Show that a recursive sequence converges

  • Thread starter Thread starter Mr Davis 97
  • Start date Start date
  • Tags Tags
    Sequence
Click For Summary
The discussion centers on proving that the recursive sequence defined by a_0 = y and a_n = (x/a_{n-1} + a_{n-1})/2 is decreasing and converges to the limit √x. Participants express confusion regarding the initial condition, particularly when a_0 is less than √x, questioning if the sequence can still be considered decreasing. The argument presented shows that for n ≥ 1, a_n is always greater than or equal to √x, and that the sequence is decreasing, satisfying the conditions for convergence. The limit is confirmed to be √x through the application of the Mean Value Theorem. The conversation also touches on the indexing convention for sequences, highlighting potential ambiguities in defining the starting index.
Mr Davis 97
Messages
1,461
Reaction score
44

Homework Statement


Let ##x,y## be positive numbers. Let ##a_0 = y## and let ##a_n = \frac{(x/a_{n-1})+a_{n-1}}{2}##. Prove that ##(a_n)## is a decreasing sequence with limit ##\sqrt{x}##.

Homework Equations

The Attempt at a Solution


I'm confused about the initial condition being an arbitrary positive real number. This is because it says to prove that ##(a_n)## is a decreasing sequence, but what if ##0<y< \sqrt{x}##? Then ##(a_n)## can't be decreasing...
 
Physics news on Phys.org
You are right.
It should be a "converging" sequence.
Or perhaps they should exempt ##a_0## from the sequence.
 
What you can show is ##a_n \ge \sqrt x## for ##n\ge 1## regardless whether or not ##a_0 = y < \sqrt x##, and that ##a_{n+1}\le a_n## for ##n\ge 1##.
 
LCKurtz said:
What you can show is ##a_n \ge \sqrt x## for ##n\ge 1## regardless whether or not ##a_0 = y < \sqrt x##, and that ##a_{n+1}\le a_n## for ##n\ge 1##.
Here is my argument:

(1) WTS: ##\forall n \ge 1##, ##a_n \ge \sqrt{x}##.

Note that it is true that ##(x - y^2)^2 \ge 0##. This implies that ##x^2-2xy^2+y^4 \ge 0 \implies x^2 + y^4 \ge 2y^2x \implies x+y^2 \ge 2y\sqrt{x} \implies \frac{\frac{x}{y} + y}{2} \ge \sqrt{x} \implies a_1 \ge \sqrt{x}##. So the base case is satisfied. Now, suppose that for some ##k \in \mathbb{N}## we have that ##a_k \ge \sqrt{x}##. Then ##a_{k+1} = \frac{\frac{x}{a_k} + a_k}{2} \ge \frac{\frac{x}{\sqrt{x}} + \sqrt{x}}{2} = \sqrt{x}##. This completes the induction.

(2) WTS: ##\forall n \ge 1##, ##a_n \ge a_{n+1}##. We see that ##a_{n+1} = \frac{\frac{x}{a_n}+a_n}{2} \le \frac{\frac{a^2_n}{a_n}+a_n}{2} = a_n##.

So we see that ##(a_n)## is bounded below and is decreasing, so by the MCT it has a limit, L. Then ##L = \frac{\frac{x}{L}+L}{2} \implies L = \sqrt{x}##.
 
Mr Davis 97 said:
Let ##a_0 = y##
What is the convention used by your text materials for the first index of a sequence? Is it 1? - or 0? - can it be either?

A nice legality could be that ##a_0## is defined as a member of a set, but it is not a member of the sequence if the text materials insist the first index of a sequence must be 1.
 
Question: A clock's minute hand has length 4 and its hour hand has length 3. What is the distance between the tips at the moment when it is increasing most rapidly?(Putnam Exam Question) Answer: Making assumption that both the hands moves at constant angular velocities, the answer is ## \sqrt{7} .## But don't you think this assumption is somewhat doubtful and wrong?

Similar threads

  • · Replies 1 ·
Replies
1
Views
2K
Replies
4
Views
2K
  • · Replies 8 ·
Replies
8
Views
2K
  • · Replies 2 ·
Replies
2
Views
2K
  • · Replies 2 ·
Replies
2
Views
2K
  • · Replies 3 ·
Replies
3
Views
1K
  • · Replies 7 ·
Replies
7
Views
2K
  • · Replies 34 ·
2
Replies
34
Views
3K
Replies
6
Views
3K
  • · Replies 9 ·
Replies
9
Views
2K